mercredi 27 février 2013


The Fourier transform





The Fourier transform takes an input function f (in red) in the “time domain” and converts it into a new function f-hat (in blue) in the “frequency domain”.In other words, the original function can be thought of as being “amplitude given time”, and the Fourier transform of the function is “amplitude given frequency”.Shown here, a simple 6-component approximation of the square wave is decomposed (exactly, for simplicity) into 6 sine waves. These component frequencies show as very sharp peaks in the frequency domain of the function, shown as the blue graph. In practice, these peaks are never that sharp. That would require infinite precision.I’m not too happy with this one yet. I might add a few frames to smooth a few steps out.




mercredi 20 février 2013

samedi 16 février 2013






















Dans tout ce chapitre : K=\mathbb{R} \text{ ou } \mathbb{C} 

I. Ensembles dénombrables

Définition :

Un ensemble D est dit dénombrable ssi il est équipotent à \mathbb{N}
Il est dit au plus dénombrable ssi il est équipotent à une partie de \mathbb{N}.
Rappels : 
* Deux ensembles E et F sont dits équipotents s'il existe une bijection f : E \longrightarrow F
* Si \mathfrak{E} est un ensemble d'ensembles, alors la relation d'équipotence sur \mathfrak{E} est une relation d'équivalence. 

Exemples : 
1) Soit \mathfrak{P} (resp. \mathfrak{J}) l'ensemble des entiers naturels pairs (resp. impairs). 
Les applications \begin{array}{rcl} f: & \mathbb{N} & \longrightarrow& \mathfrak{P}&\\  & n & \longrightarrow & 2n \end{array}     et     \begin{array}{rcl}f: & \mathbb{N} & \longrightarrow& \mathfrak{J}&\\  & n & \longrightarrow & 2n+1 \end{array} sont des bijections. 
Donc \mathfrak{P} et \mathfrak{J} sont dénombrables. 
2) Tout ensemble fini est au plus dénombrable car il est équipotent à une partie de \mathbb{N} de la forme \ldbrack1,n\rdbrack \: \left(n \in \mathbb{N}\right). 

Remarque : 
Si D est un ensemble dénombrable (resp. au plus dénombrable) alors tout ensemble \Delta équipotent à D est dénombrable (resp. au plus dénombrable).
Théorème :

Soit P une partie infinie de \mathbb{N}. Alors : 
* P est dénombrable. 
* Soit \sigma : \mathbb{N} \longrightarrow P tel que : \lbrace {\sigma(0)=\min P\atop \forall n>0 , \sigma(n)=\min(P\backslash \lbrace \sigma(0),\cdots , \sigma(n-1) \rbrace ) } 
Alors \sigma est l'unique bijection strictement croissante de \mathbb{N} dans P.
Corollaire :

Soit D un ensemble. 
Alors D est au plus dénombrable ssi D est fini ou dénombrable.
Proposition :

Soit D un ensemble. 
* S'il existe une application injective f:D\longrightarrow \mathbb{N}, alors D est au plus dénombrable. 
* S'il existe une application surjective g: \mathbb{N} \longrightarrow D, alors D est au plus dénombrable.
Exemple : 
Soit \begin{array}{rcl}f: & \mathbb{N}^2 & \longrightarrow& \mathbb{N}&\\  & (n,m) & \longrightarrow & 2^n\times3^m \end{array} 
f est injective par unicité de la décomposition d'un entier en produit de facteurs premiers. 
\mathbb{N}^2 est au plus dénombrable, et comme il est infini, il est dénombrable.
Théorème :

Toute réunion d'une suite d'ensembles dénombrables est un ensemble dénombrable. 
Toute réunion d'une suite d'ensembles au plus dénombrables est un ensemble au plus dénombrable.
Théorème :

Soit D un ensemble. Alors les propositions suivantes sont équivalentes : 
* D est au plus dénombrable. 
* Il existe une suite (D_n)_{n\in\mathbb{N}} de parties finies de D tq :  \lbrace {(\forall n\in\mathbb{N}) \: : \: D_n \subset D_{n+1}\atop \bigcup_{n\in\mathbb{N}} D_n = D}
Exemple : 
D = \mathbb{Z} 
D_n = \ldbrack-n,n\rdbrack (avec : n \in \mathbb{N}^*
On a : 
    * \mathbb{Z} = \bigcup_{n\in\mathbb{N}} D_n 
    * D_n \subset\mathbb{Z} , D_n fini. 
    * D_n \subset D_{n+1} 
\mathbb{Z} est au plus dénombrable et puisque \mathbb{Z} est infini, \mathbb{Z} est dénombrable. 

II. Familles sommables de réels positifs

I est un ensemble au plus dénombrable.
Définition :

Soit (a_i)_{i\in I} une famille de réels positifs indexée par I
On dit que (a_i)_{i\in I} est sommable ssi \rm \lbrace  \displaystyle\sum_{i\in J}a_i / J\subset I, J fini\rbrace  est majoré dans \mathbb{R}
Dans ce cas, la borne supérieure de cet ensemble est appelée la somme de la famille (a_i)_{i\in I} et notée \displaystyle \sum_{i\in I} a_i.
Notation : 
\mathfrak{F}(I) désigne l'ensemble de toutes les parties finies de I
Ainsi : \displaystyle \sum_{i\in I} a_i = \sup_{J\in \mathfrak{F}(I)} \left(\displaystyle \sum_{i\in J} a_i\right) si la famille est sommable. 

Exemples : 
Exemple 1 : Soit I = \mathbb{N}^*^2 \, , \, (i,j)\in I \, , \, a_{ij}=\frac{1}{i^2\times j^2} 
Soit J\in\mathfrak{F}(I) ; il existe N\in\mathbb{N}^* tq : J \subset \ldbrack1,N\rdbrack^2 
En effet, soit (i,j) \in Jk = (i,j)\in\mathbb{N}^{*}^2 = \bigcup_{n\in\mathbb{N}^*} \ldbrack1,n\rdbrack^2
\exists n_k \in \mathbb{N} tq k \in \ldbrack1,n_k\rdbrack^2 
Soit N = \max_{k\in J}( n_k)N existe car J est fini. 
On a k = (i,j)\in J
On a : \lbrace {1 \leq i \leq n_k \leq N\atop 1\leq j \leq n_k \leq N} d'où : k=(i,j) \in \mathbb{[\mathbb{1},N|]^2 
On conclut : J\subset \ldbrack1,N\rdbrack^2 (ce qui fallait démontrer) 

Soit maintenant N un tel entier : 
\displaystyle \sum_{(i,j)\in \ldbrack1,N\rdbrack^2} \frac{1}{i^2 \times j^2} =  \displaystyle \sum_{i=1}^{N} \left(\displaystyle \sum_{j=1}^{N} \frac{1}{i^2\times j^2}\right)   = \displaystyle \sum_{i=1}^{N} \frac{1}{i^2} \left(\displaystyle \sum_{j=1}^{N} \frac{1}{j^2}\right)   = \left(\displaystyle \sum_{i=1}^{N} \frac{1}{i^2}\right) \left(\displaystyle \sum_{j=1}^{N} \frac{1}{j^2}\right)   = \displaystyle \left(\sum_{i=1}^{N} \frac{1}{i^2}\right)^2, de plus, on sait que \displaystyle \sum_{n\geq 1}  \frac{1}{n^2} converge. 
Posons : S sa somme. \forall J\in \mathfrak{F}(I)\displaystyle \sum_{(i,j)\in J} a_{ij}\leq S^2
Donc : (a_{ij})_{(i,j)\in I} est sommable et \displaystyle \sum_{(i,j)\in J} a_{ij}\leq S^2
Montrons que : S^2 = \displaystyle \sum_{(i,j)\in I} a_{ij} = \sup_{J\in \mathfrak{F}(I)} \sum_{(i,j)\in J} a_{ij}
S^2 est déjà un majorant de l'ensemble : A=\lbrace \displaystyle \sum_{(i,j)\in J} a_{ij} \, / \, J\in\mathfrak{F}(I)\rbrace
Soit \epsilon > 0 \, ; \, \left(\displaystyle \sum_{i=1}^{n} \frac{1}{i^2}\right)^2 \longrightarrow_{n\to+\infty}S^2, et comme \left(\left(\displaystyle \sum_{i=1}^{n} \frac{1}{i^2}\right)^2\right)_n est croissante. 
Alors : S^2 = \sup \left(\displaystyle \sum_{i=1}^{N} \frac{1}{i^2}\right)^2 
Posons donc : N \in \mathbb{N}^* tq : S^2 - \epsilon < \left(\displaystyle \sum_{i=1}^{N} \frac{1}{i^2}\right)^2, d'où S^2 - \epsilon < \displaystyle \sum_{i=1}^{n} \displaystyle \sum_{j=1}^{n} \frac{1}{i^2\times j^2} 
C'est-à-dire : S^2 - \epsilon < \displaystyle \sum_{(i,j) \in \ldbrack1,N\rdbrack^2} a_{ij} \in A . 
Donc : \displaystyle \sum_{(i,j)\in \mathbb{N}^{*}^{2}} \frac{1}{i^2\times j^2} = S^2


Exemple 2 : I=\mathbb{Z} \, , \, a_i = e^{-i} 
Soit J_n = \ldbrack-n,0\rdbrack \, , \, J_n \in \mathfrak{F}(\mathbb{Z}) 
On a : \displaystyle \sum_{i\in J_n} a_i = \sum_{i=-n}^{0} e^{-i} = \sum_{i=0}^n e^i \longrightarrow_{n\to+\infty} +\infty 
Donc : A=\lbrace \displaystyle \sum_{i\in J} a_i \: / \: J\in\mathfrak{F}(\mathbb{Z})\rbrace  n'est pas majoré. 
Donc (e^{-i})_{i\in\mathbb{Z}} n'est pas sommable. 

Remarques : 
1) Soit (a_i)_i une famille de réels positifs sommable, alors : \displaystyle \sum_{i\in I} a_i \geq 0
Si \displaystyle \sum_{i\in I} a_i = 0\forall J\in\mathfrak{F}(I) \: : \: 0\leq \displaystyle \sum_{i\in J} a_i \leq 0 
D'où \forall J\in\mathfrak{F}(I) \: : \: \displaystyle \sum_{i\in J} a_i = 0
En particulier : \forall j \in I \: :  \: \displaystyle \sum_{i\in\lbrace j\rbrace } a_i=0, ie : \forall j \in I \:  : \: a_j=0
Réciproquement, si la famille est nulle, elle est sommable de somme 0. 
Conclusion : \displaystyle \sum_{i\in I} a_i = 0 ssi \forall i \in I \, , \, a_i = 0
2) Toute sous famille d'une famille de réels positifs sommable est elle-même sommable de somme plus petite. 
Théorème :

Soit (a_i)_{i\in I} et (b_i)_{i\in I} deux familles de réels positifs tq : \forall i \in I \: : \: a_i \leq b_i. Alors : 
* Si (b_i)_{i\in I} est sommable, il en est de même de (a_i)_{i\in I} et on a : \displaystyle \sum_{i\in I} a_i \leq \displaystyle \sum_{i\in I} b_i
* Si (a_i)_{i\in I} n'est pas sommable, il en est de même de (b_i)_{i\in I}.
Exemples : 
Exemple 1 : I = \mathbb{N}^{*}^2 \, , \, a_{ij} = \frac{2}{i^4+j^4} 
Pour tout (i,j)\in \mathbb{N}^*^2, on a : i^4+j^4 \geq 2i^2\times j ^2 
D'où : a_{ij} \leq \frac{1}{i^2\times j^2} = b_{ij} 
Comme (b_{ij})_{(i,j)\in \mathbb{N}^*^2} est sommable (d'après un exemple précédent), on a : (a_{ij})_{(i,j)\in \mathbb{N}^*^2} est sommable. 

Exemple 2 : \left(\frac{ln(i+2)}{i}\right)_{i\in\mathbb{N}^*} 
On a : \frac{1}{i} \leq \frac{\ln(i+2)}{i} \: \forall i \in \mathbb{N}^*
\left(\frac{1}{i}\right)_{i\in \mathbb{N}^*} est non sommable. 
On en déduit que : \left(\frac{\ln(i+2)}{i}\right)_{i\in \mathbb{N}^*} est non sommable.
Proposition :

Soit (a_i)_{i\in I} et (b_i)_{i\in I} deux familles de réels positifs indexée par I sommables. Soit c\in \mathbb{R}. Alors : 
* (a_i+b_i)_{i\in I} est sommable et \displaystyle \sum_{i\in I} (a_i+b_i) = \displaystyle \sum_{i\in I} (a_i) + \displaystyle \sum_{i\in I} (b_i)
* (ca_i)_{i\in I} est sommable et \displaystyle \sum_{i\in I} ca_i = c \displaystyle \sum_{i\in I} a_i.
Théorème :

Soit (J_n)_{n\in \mathbb{N}} une suite croissante de parties finies de I tq \bigcup_{n\in \mathbb{N}}  J_n= I
Soit (a_i)_{i\in I} une famille de réels positifs. Alors les propositions suivantes sont équivalentes : 
* (a_i)_{i\in I} est sommable. 
* La suite réelle \left(\displaystyle \sum_{i\in J_n} a_i\right)_n est majorée. 
* La suite réelle \left(\displaystyle \sum_{i\in J_n} a_i\right)_n est convergente. 
De plus, dans ce cas : \displaystyle \sum_{i\in I} a_i = \sup_{n\in\mathbb{N}}\left(\displaystyle \sum_{i\in J_n}a_i\right) = \displaystyle \lim_{n\to +\infty} \left(\displaystyle \sum_{i\in J_n}a_i\right)
Exemple : 
I = \mathbb{N}^*^2 \, , \, a_{ij}=\frac{1}{(i+j)^r} avec : r \in \mathbb{R} donné. 
Soit J_n=\lbrace (i,j)\in\mathbb{N}^*^2 \, / \, i+j\leq n\rbrace . On a : 
* (J_n) est croissante. 
* \forall n \, , \, J_n \in \mathfrak{F}(\mathbb{N}^*^2) et \bigcup_{n} J_n = \mathbb{N}^*^2 
\displaystyle \sum_{(i,j)\in J_n} a_{ij} = \displaystyle \sum_{k=2}^n \left(\displaystyle \sum_{(i,j)\in\mathbb{N}^*^2\atop i+j=k} \frac{1}{(i+j)^r}\right) = \displaystyle \sum_{k=2}^n \frac{1}{k^r}\left(\displaystyle \sum_{(i,j)\in\mathbb{N}^*^2\atop i+j=k} 1\right) = \displaystyle \sum_{k=2}^n \frac{1}{k^r} .Card\lbrace (i,j)\in \mathbb{N}^*^2 / i+j=k\rbrace  = \displaystyle \sum_{k=2}^n \frac{k-1}{k^r} = \displaystyle \sum_{k=1}^n \frac{k-1}{k^r}
Or, \displaystyle \frac{n-1}{n^r} \sim_{n\to+\infty} \frac{1}{n^{r-1}} 
Donc : (a_{ij})_{(i,j)} sommable \Longleftrightarrow \displaystyle \sum \frac{n-1}{n^r} converge \Longleftrightarrow r - 1 > 1 \Longleftrightarrow r > 2 . 
En outre dans ce cas : \displaystyle \sum_{(i,j)\in\mathbb{N}^*^2} \frac{1}{(i+j)^r} = \displaystyle \lim_{n\to+\infty} \displaystyle \sum_{(i,j)\in J_n} \frac{1}{(i+j)^r} = \displaystyle \lim_{n\to+\infty} \displaystyle \sum_{k=1}^n \frac{k-1}{k^r} = \displaystyle \sum_{k=1}^{+\infty} \frac{k-1}{k^r}
Soit : \zeta : x \longrightarrow \displaystyle \sum_{n=1}^{+\infty} \frac{1}{n^x}D_{\zeta} = ]1,+\infty[ d'après la règle de Riemann. 
Alors : \displaystyle \sum_{(i,j)\in\mathbb{N}^*^2} \frac{1}{(i+j)^r} = \displaystyle \sum_{k=1}^{+\infty} \left(\frac{1}{k^{r-1}} -\frac{1}{k^r} \right) = \displaystyle \sum_{k=1}^{+\infty} \frac{1}{k^{r-1}} - \displaystyle \sum_{k=1}^{+\infty} \frac{1}{k^{r}}  = \zeta(r-1) - \zeta(r)
La fonction \zeta est appelée la fonction Zéta de Riemann

Rappel : 
On dit que (J_n) est une suite exhaustive de \mathbb{N} ssi : 
* J_n est une partie finie de \mathbb{N}
* J_n\subset J_{n+1}
* \bigcup J_n = \mathbb{N}.
Proposition :

Soit (a_n)_{n\in\mathbb{N}} une suite de réels positifs. 
Alors (a_n)_{n\in\mathbb{N}} est sommable ssi la série \displaystyle \sum_{n\geq 0 } a_n converge . 
De plus, dans ce cas : \displaystyle \sum_{n\in\mathbb{N}} a_n = \displaystyle \sum_{n=0}^{+\infty} a_n.
Proposition :

Soit (a_i)_{i\in I} une famille dénombrable de réels positifs, soit \sigma : \mathbb{N} \longrightarrow I une bijection . 
Alors (a_i)_{i\in I} est sommable ssi \displaystyle \sum_{n\geq 0 } a_{\sigma(n)} converge. 
De plus, dans ce cas : \displaystyle \sum_{i\in I} a_i = \displaystyle \sum_{n=0}^{+\infty} a_{\sigma(n)}.
Corollaire :

Soit (a_n)_{n\in\mathbb{N}} une suite de réels positifs, soit \sigma une permutation de \mathbb{N}
On a alors : \displaystyle \sum_{n\geq 0 } a_n converge ssi \displaystyle \sum_{n\geq 0 } a_{\sigma(n)}, et dans ce cas : \displaystyle \sum_{n=0}^{+\infty} a_n = \displaystyle \sum_{n=0}^{+\infty} a_{\sigma(n)}.

III. Famille sommable d'éléments de K (cas général)

1. Sommabilité

Définition :

Soit (a_i)_{i\in I} une famille d'éléments de K indexée par I
On dit que (a_i)_{i\in I} est sommable ssi (|a_i|)_{i\in I} est sommable en tant que famille de réels positifs.
Exemples : 
1) I = \mathbb{N}^*^2 \, ; \, a_{ij}=\frac{(-1)^i}{(i+j)^}   (r>2) 
|a_{ij}| = \frac{1}{(i+j)^r} et r>2 ; d'après un exemple précédent : (a_{ij})_{(i,j)\in\mathbb{N}^*^2} est sommable. 
2) I = \mathbb{Z}^*  \, ; \, a_n = \frac{e^{in}}{n^2} 
On a : |a_n|=\frac{1}{n^2}, donc (a_n)_{n\in\mathbb{Z}^*} est sommable. 

Remarques : 
* Toute famille finie d'éléments de K est sommable. 
* Toute famille presque nulle d'éléments de K est sommable. 
* Toute sous famille d'une famille sommable d'éléments de K est, elle-même sommable.
Proposition :

Soit (a_i)_{i\in I} une famille d'éléments de K et (b_i)_{i\in I} une famille de réels positifs indexée par le même I tq : 
(\forall i \in I) \: : \: |a_i|\leq b_i. Alors : 
* Si (b_i)_{i\in I} est sommable, il en est de même de (a_i)_{i\in I}
* Si (a_i)_{i\in I} est non sommable, il en est de même de (b_i)_{i\in I}.
Remarque : 
On note \mathfrak{l}^1(I) l'ensemble des familles sommables d'éléments de K indexés par I
* \mathfrak{l}^1(I) \not = \not O car il contient la famille nulle. 
* \mathfrak{l}^1(I) \subset K^I
* \mathfrak{l}^1(I) est stable par combinaison linéaire. 
* \mathfrak{l}^1(I) est un K-ev, sev de K^I.
Proposition :

Soit (z_p)_{p\in I} une famille des nombres complexes indexée par I et soit a_p = \mathfrak{Re}(z_p) et b_p = \mathfrak{Im}(z_p)
Alors (z_p)_{p\in I} est sommable ssi (a_p)_{p\in I} et (b_p)_{p\in I} sont sommables.

2. Somme d'une famille sommable

Théorème - Définition :

Soit (a_i)_{i\in I} une famille sommable d'éléments de K. Soit (J_n)_{n\in \mathbb{N}} une suite exhaustive de I, alors : 
* La suite (\displaystyle \sum_{i\in J_n} a_i ) est convergente dans K
* La limite de cette suite ne dépend pas du choix de (J_n)_{n\in \mathbb{N}}, on l'appelle la somme de la famille (a_i)_{i\in I}, on la note \displaystyle \bigsum_{i\in I} a_i.
Remarque : 
En général : (a_i)_{i\in I} sommable \Rightarrow \left(\displaystyle \sum_{i\in J_n} a_i\right) converge. 
La réciproque est fausse en général . 

Contre-exemple : 
Soit I = \mathbb{N} \, , \, a_i = \frac{(-1)^i}{i+1} et J_n = \ldbrack0,n\rdbrack 
(J_n) est exhaustive de \mathbb{N}
\left(\displaystyle \sum_{i\in J_n} a_i \right) = \left(\displaystyle \sum_{i=0}^N \frac{(-1)^1 }{i+1}\right) converge (critère des séries alternées). 
Or, \left(|a_i|\right)_{i\in\mathbb{N}} n'est pas sommable car \sum \frac{1}{n+1} diverge. 
Donc (a_i)_{i\in\mathbb{N}} n'est pas sommable.
Proposition :

Soit (a_i)_{i\in I} et (b_i)_{i\in I} deux familles sommables d'éléments de K et soit \lambda \in\mathbb{R}. Alors : 
    * (\lambda a_i + b_i)_{i\in I} est sommable. 
    * \displaystyle \sum_{i\in I} \left(\lambda a_i+b_i\right) = \lambda \displaystyle \sum_{i\in I} a_i + \displaystyle \sum_{i\in I} b_i
Proposition :

Soit (z_k)_{k\in \mathbb{N}} une famille de nombres complexes. On pose : a_k = \mathfrak{Re}(z_k) et b_k = \mathfrak{Im}(z_k). Alors : 
    * (z_k)_{k\in \mathbb{N}} est sommable ssi (a_k)_{k \in \mathbb{N}} et (b_k)_{k\in \mathbb{N}} sont sommables. 
    * Si (z_k)_{k\in \mathbb{N}} est sommable, on a : \displaystyle \sum_{k\in I} a_k = \displaystyle \sum_{k\in I} a_k + i \displaystyle \sum_{k\in I} b_k
Proposition :

Soit (a_i)_{i\in I} une famille sommable d'éléments de K. Alors : 
|\displaystyle \sum_{i\in I} a_i | \leq \displaystyle \sum_{i\in I} |a_i|
Théorème :

Soit (a_i)_{i\in I} et (b_i)_{i\in I} deux familles d'éléments de K tq : (a_i ^2)_{i\in I} et (b_i ^2)_{i\in I} sont sommables. 
Alors : (\bar{a_i} b_i)_{i\in I} est sommable. 
De plus, dans ce cas : |\displaystyle \sum_{i\in I} \bar{a_i} b_i | \leq \left(\displaystyle \sum_{i\in I} |a_i|^2\right)^{1/2}\left(\displaystyle \sum_{i\in I} |b_i|^2\right)^{1/2}
Théorème :

Soit (a_n)_{n\in \mathbb{N}} une suite d'éléments de K. Alors la suite (a_n)_{n\in \mathbb{N}} est sommable ssi la série \displaystyle \sum_{n\geq 0} a_n converge absolument. 
De plus, dans ce cas : \displaystyle \sum_{n\in\mathbb{N}} a_n = \displaystyle \sum_{n=0}^{+\infty} a_n.
Proposition :

Soit (a_i)_{i\in I} une famille d'éléments de K, soit \sigma : \mathbb{N} \longrightarrow I une bijection. 
Alors (a_i)_{i\in I} est sommable ssi : \displaystyle \sum_{n\geq 0} a_{\sigma(n)} converge absolument. 
De plus, dans ce cas : \displaystyle \sum_{i\in I} a_i = \displaystyle \sum_{n=0}^{+\infty} a_{\sigma(n)}
Corollaire :

Soit (a_n)_{n\in \mathbb{N}} une suite d'éléments de K et \sigma une permutation de \mathbb{N}. Alors : 
\displaystyle \sum_{n\geq 0 } a_n converge absolument ssi \displaystyle \sum_{n\geq 0} a_{\sigma(n)} converge absolument. 
De plus, dans ce cas : \displaystyle \sum_{n=0}^{+\infty} a_n = \displaystyle \sum_{n=0}^{+\infty} a_{\sigma(n)}.


IV. Suites doubles sommables

Théorème :

Soit \displaystyle \sum_{n\geq 0 } a_n et \displaystyle \sum_{n\geq 0 } b_n deux séries absolument convergentes. 
Alors la suite double (a_n b_m)_{(n,m)\in\mathbb{N}^2} est sommable et on a : \displaystyle \sum_{(n,m)\in\mathbb{N}^2} a_n b_m = \left(\displaystyle \sum_{n=0}^{+\infty} a_n\right)\left(\displaystyle \sum_{n=0}^{+\infty} b_n\right).
Théorème : "d'interversion des sommations"

Soit (a_{ij})_{(i,j)\in\mathbb{N}^2} une suite double de réels positifs. Alors les propositions suivantes sont équivalentes : 
    * (a_{ij})_{(i,j)\in\mathbb{N}^2} est sommable. 
    * \lbrace  \forall i\in\mathbb{N} \, , \, \displaystyle \sum_{j\geq 0}a_{ij} \text{ convege. } \\ \displaystyle \sum_{i\geq 0} \left(\displaystyle \sum_{j=0}^{+\infty} a_{ij}\right) \: \text{ converge.} \. 
    * \lbrace  \forall j\in\mathbb{N} \, , \, \displaystyle \sum_{i\geq 0}a_{ij} \text{ convege. } \\ \displaystyle \sum_{j\geq 0} \left(\displaystyle \sum_{i=0}^{+\infty} a_{ij}\right) \text{ converge.} \. 
De plus, dans ce cas : \displaystyle \sum_{(i,j) \in \mathbb{N}^2} a_{ij} = \displaystyle \sum_{i=0}^{+\infty} \left(\displaystyle \sum_{j=0}^{+\infty}a_{ij}\right) = \displaystyle \sum_{j=0}^{+\infty} \left(\displaystyle \sum_{i=0}^{+\infty}a_{ij}\right).
Théorème de "Fubini" :

Soit (a_{ij})_{(i,j) \in \mathbb{N}^2} une suite double d'éléments de K. Alors si (a_{ij})_{(i,j)\in\mathbb{N}^2} est sommable, on a : 
    * \lbrace  \forall i\in\mathbb{N} \, , \, \displaystyle \sum_{j\geq 0}a_{ij} \text{ convege absolument.} \\ \displaystyle \sum_{i\geq 0} \left(\displaystyle \sum_{j=0}^{+\infty} a_{ij}\right) \text{ converge absolument.} \. 
    * \lbrace  \forall j \in\mathbb{N} \, , \, \displaystyle \sum_{i\geq 0}a_{ij} \text{ converge absolument.} \\ \displaystyle \sum_{j\geq 0} \left(\displaystyle \sum_{i=0}^{+\infty}a_{ij}\right) \text{ converge absolument.} \. 
    * \displaystyle \sum_{(i,j) \in \mathbb{N}^2} a_{ij} = \displaystyle \sum_{i=0}^{+\infty} \left(\displaystyle \sum_{j=0}^{+\infty}a_{ij}\right) = \displaystyle \sum_{j=0}^{+\infty} \left(\displaystyle \sum_{i=0}^{+\infty}a_{ij}\right).
Remarque : 
La réciproque du théorème de Fubini est fausse en général. 

Exemple : 
Soient a,b \in\mathbb{C} \, ; \, |U_{ij}| = \frac{a^ib^j}{(i+j)!} 
On a : |U_{ij}| \leq \frac{|a|^i}{i!}\frac{|b|^j}{j!} 
On sait que : \displaystyle \sum_{n\geq 0}\frac{|a|^n}{n!} \text{ et } \displaystyle \sum_{n\geq 0}\frac{|b|^n}{n!} sont deux séries à termes positifs convergentes. 
Donc (U_{ij})_{(i,j)\in\mathbb{N}^2} est sommable. 
\displaystyle \sum_{(i,j)\in\mathbb{N}^2} \frac{a^ib^j}{(i+j)!} = \displaystyle \lim_{n\to+\infty} \displaystyle \sum_{(i,j)\in A_n} \frac{a^ib^j}{(i+j)!} avec A_n = \lbrace (i,j)\in\mathbb{N}^2/i+j\leq n\rbrace
Donc : \displaystyle \sum_{(i,j)\in\mathbb{N}^2} \frac{a^ib^j}{(i+j)!} = \displaystyle \lim_{n\ti+\infty} \displaystyle \sum_{k=0} \left(\displaystyle \sum_{(i,j)\in\mathbb{N}^2/ i+j = k} \frac{a^ib^j}{(i+j)!}\right) = \displaystyle \lim_{n\ti+\infty} \displaystyle \sum_{k=0}^n \frac{1}{k!} \left(\displaystyle \sum_{(i,j)\in\mathbb{N}^2/ i+j=k} a^ib^j\right) 
      * Si a \not= b : 
On a : b^{k+1} - a^{k+1} = (b - a) \displaystyle \sum_{i=0}^k a^i b^{k-i} = (b - a) \displaystyle \sum_{(i,j)\in\mathbb{N}^2 / i+j=k} a^i b^j 
\displaystyle \sum_{(i,j)\in\mathbb{N}^2} \frac{a^ib^j}{(i+j)!} = \displaystyle \lim_{n\to +\infty} \displaystyle \sum_{k=0}^n\frac{b^{k+1}-a^{k+1}}{(b-a)k!} = \frac{1}{b-a}\displaystyle \lim_{n\to +\infty} \displaystyle \sum_{k=0}^n \frac{b^{k+1}-a^{k+1}}{k!} = \frac{be^b-ae^a}{b-a}
      * Si a = b : 
\displaystyle \sum_{(i,j)\in\mathbb{N}^2} \frac{a^i b^j}{(i+j)!} = \displaystyle \sum_{(i,j)\in\mathbb{N}^2} \frac{a^{i +j}}{(i+j)!} = (a+1)e^a

Contre-exemple : 
Considérons la famille (u_{m,n})_{(m,n)\in\mathbb{N}\times\mathbb{N}} définie par u_{m,n} = \frac{1}{m^2-n^2} si m \neq n et u_{n,n} = 0
Il est clair que pour m fixé la série \displaystyle \sum_{n\in\mathbb{N}} u_{m,n} est convergente et que pour n fixé la série \displaystyle \sum_{m\in\mathbb{N}} u_{m,n} est convergente. 
On a \displaystyle \sum_{n=0}^\infty u_{0,n} = \displaystyle \sum_{n=1}^\infty\frac{1}{n^2} < 0 
Soit m > 0. Comme \frac{1}{m^2-n^2} = \frac{1}{2m(m+n)}+\frac{1}{2m(m-n)} et comme \frac{1}{2m(m+n)} + \frac{1}{2m(m-(2m+n))} = 0, on voit que \displaystyle \sum_{n=0}^\infty u_{m,n} = -\frac{3}{4m^2} car dans la somme les seuls termes qui ne s'annulent pas deux à deux sont \frac{-1}{2m^2} et \frac{-1}{4m^2}. Ceci montre que la série \displaystyle \sum_{m\in\mathbb{N}} \left(\displaystyle \sum_{n=0}^\infty u_{m,n}\right)est convergente et que \displaystyle \sum_{m=0}^\infty \left(\displaystyle \sum_{n=0}^\infty u_{m,n}\right) < 0
Mais on a u_{m,n} = -u_{n,m} pour tout couple (m,n), donc \displaystyle \sum_{n=0}^\infty \left(\displaystyle \sum_{m=0}^\infty u_{m,n}\right) > 0 et on voit que dans ce cas 
\displaystyle \sum_{m=0}^\infty \left(\displaystyle \sum_{n=0}^\infty u_{m,n}\right) \neq \displaystyle \sum_{n=0}^\infty \left(\displaystyle \sum_{m=0}^\infty u_{m,n}\right) 

V. Groupements de termes

Les séries envisagées dans ce paragraphe sont à termes dans un evn E.
Définitions générales :

Soient \displaystyle \sum_{n\geq 0} u_n une série et \sigma : \mathbb{N}\longrightarrow \mathbb{N} une extractrice (application linéaire strictement croissante). Notons pour tout n\in\mathbb{N} \: : \: v_n = \displaystyle \sum_{k = \sigma(n)}^{\sigma(n+1)-1} u_k
On dit que la série \displaystyle \sum_{n\geq 0} v_n a été obtenue par groupement de termes à partir de la série \displaystyle \sum_{n\geq 0} u_n
Les v_n sont appelés les paquets et \sigma(n+1) - \sigma(n) est appelée la longueur du paquet v_n.
Nous nous intéressons ici aux liens éventuels entre les natures de \displaystyle \sum_{n\geq 0} u_n et \displaystyle \sum_{n\geq 0} v_n et, dans le cas de convergence, aux liens éventuels entre leurs sommes.
Proposition :

Si \displaystyle \sum_{n\geq 0} u_n converge, alors \displaystyle \sum_{n\geq 0} v_n converge et : \displaystyle \sum_{n=0}^{+\infty} v_n = \displaystyle \sum_{k=\sigma(0)}^{+\infty} u_k.
Remarque : 
* La réciproque de cette proposition est fausse en général. 
* Cette proposition n'est pas pratique car elle suppose que \displaystyle \sum_{n\geq 0} u_n converge déjà.
Théorème : "de groupement de termes" :

Avec les notations précédentes : 
Si \lbrace  u_n \longrightarrow_{n\to+\infty} 0 \\ \left(\sigma(n+1) - \sigma(n)\right)_{n\in\mathbb{N} \text{ est born\bar{e}e} \/ , alors les séries \displaystyle \sum_{n\geq 0} u_n et \displaystyle \sum_{n\geq 0} v_n sont de même nature et, dans le cas de convergence, on a : \displaystyle \sum_{n=0}^{+\infty} v_n  = \displaystyle \sum_{k=\sigma(0)}^{+\infty} u_k.
Remarque : 
Si la longueur des paquets n'est pas bornée, il se peut que \displaystyle \sum_{n\geq 0} u_n diverge et \displaystyle \sum_{n\geq 0} v_n converge. 

Exemple : 
\displaystyle \sum_{n\geq 0} u_n = 1 - \frac{1}{2} - \frac{1}{2} + \frac{1}{3} + \frac{1}{3} + \frac{1}{3} - \frac{1}{4} - \frac{1}{4} - \frac{1}{4} - \frac{1}{4} + \cdots  
\displaystyle \sum_{n\geq 0} u_n diverge d'après la proposition précédente, puisque la série groupée ainsi : 
1 + \left(-\frac{1}{2} - \frac{1}{2}\right) + \left(\frac{1}{3} + \frac{1}{3} + \frac{1}{3}\right) - \cdots diverge. 
Et \displaystyle \sum_{n\geq 0} v_n = \left(1 - \frac{1}{2} - \frac{1}{2}\right) + \left(\frac{1}{3} + \frac{1}{3} + \frac{1}{3} - \frac{1}{4} - \frac{1}{4} - \frac{1}{4} - \frac{1}{4}\right) + \cdots qui converge.




Prérequis : Suites et séries de fonctions.



I. Définitions

* \mathbb{K} = \mathbb{R} \text{ ou } \mathbb{C}
* Étant donné une suite (a_n)_{n \in \mathbb{N}} de nombres complexes, on lui associe la série de fonctions \displaystyle \sum_{n \geq 0} f_n où : \begin{array}{rrcl} f_n : & \mathbb{K} & \longrightarrow & \mathbb{C} \\  & z & \mapsto & a_n z^n \end{array} 
* \displaystyle \sum_{n \geq 0} f_n est dite la série entière associée à (a_n)_{n \in \mathbb{N}} dont elle est appelée la suite des coefficients
* \displaystyle \sum_{n \geq 0} f_n est dite série entière de la variable réelle si \mathbb{K} = \mathbb{R}, et de la variable complexe si \mathbb{K} = \mathbb{C}

Une série entière de coefficients (a_n) se note généralement : \displaystyle \sum_{n\geq 0} a_n z^n ou \displaystyle \sum_{n \geq 0} a_n x^n


II. Convergence d'une série entière

1. Rayon de convergence

Lemme d'Abel :
Soit \displaystyle \sum a_nz ^n une série entière et r > 0 tq (a_n r^n)_n est bornée. 
Alors, pour tout z \in \mathbb{K} on a : |z| < r entraîne \displaystyle \sum_{n \geq 0} a_n z^n converge absolument.
Théorème - Définition :
Soit \displaystyle \sum a_n z^n une série entière. Alors il existe un unique nombre noté R avec R \in [0 , +\infty[ \cup \lbrace +\infty \rbrace tq : 
\forall z \in \mathbb{K} \, : \, \left \lbrace \begin{array}{llll} |z| < R & \Longrightarrow & \displaystyle \sum a_n z^n & \text{ converge absolument } \\ |z| > R & \Longrightarrow & \displaystyle \sum a_n z^n & \text{ diverge} \\ \end{array} \right. 
R s'appelle le rayon de convergence de \displaystyle \sum_{n \geq 0} a_n z^n.

Exemple : 
* \displaystyle \sum_{n \geq 0} \frac{n}{n^2+1} z^n : 
Pour |z| < 1 \, : \, \left|\displaystyle \frac{n}{n^2+1} z^n \right| = \displaystyle \frac{n}{n^2+1} |z|^n =o \left(\frac{1}{n^2}\right) ; \sum \displaystyle \frac{n}{n^2+1} z^n converge absolument. 
Pour |z| > 1 \, : \, \left|\displaystyle \frac{n}{n^2+1} z^n \right| = \displaystyle \frac{n}{n^2+1} |z|^n \xrightarrow[n\to+\infty]{} +\infty ; \sum \displaystyle \frac{n}{n^2+1} z^n diverge. 
Donc \boxed{R = 1}. 

Proposition :
Soit \displaystyle \sum_{n\geq 0} a_n z^n une série entière de rayon de convergence R. Alors : 
* R = \sup_{\bar{\mathbb{R}}} \lbrace r \in \mathbb{R}_+ / (a_n r^n) \text{ bornée } \rbrace
* R = \sup_{\bar{\mathbb{R}}} \lbrace  |z| / z \in \mathbb{K} \text{ et } \sum a_n z^n \text{ converge absolument } \rbrace
* R = \sup_{\bar{\mathbb{R}}} \lbrace  |z|  / z \in \mathbb{K} \text{ et } \sum a_nz^n \text{ converge } \rbrace
* R = \sup_{\bar{\mathbb{R}}} \lbrace  |z|  / z \in \mathbb{K} \text{ et } a_nz^n \xrightarrow[n \to +\infty]{} 0 \rbrace
* R = \sup_{\bar{\mathbb{R}}} \lbrace  |z|  / z \in \mathbb{K} \text{ et } (a_n z^n) \text{ converge } \rbrace .

Notation et vocabulaire : 
Soit \displaystyle \sum_{n\geq 0} a_n z^n une série entière de rayon de convergence R. Alors D(0 , R) = \lbrace z \in \mathbb{K} / |z| < R \rbrace est appelé le disque de convergence de \displaystyle \sum_{n\geq 0} a_n z^n
En particulier, si \mathbb{K} = \mathbb{R}D(0,R) = ]-R , R[ ; on l'appelle l'intervalle de convergence. 

Remarques : 
1) Si f est la somme de la série entière \displaystyle \sum_{n \geq 0} a_n z^n, alors : D(0 , R) \subset D_f \subset \overline{D(0 , R)}
2) Si \forall z \in \mathbb{K} : 
* \displaystyle \sum_{n\geq 0} a_n z^n converge \Longleftrightarrow \displaystyle \sum_{n \geq 0 } b_n z^n converge. 
Ou : 
* \displaystyle \sum_{n\geq 0} a_n z^n convergence absolument \Longleftrightarrow \displaystyle \sum_{n \geq 0} b_n z^n converge absolument. 
Alors : R_{cv} \left(\displaystyle \sum_{n\geq 0} a_n z^n \right) = R_{cv} \left( \displaystyle \sum_{n \geq 0} b_n z^n \right) 

Proposition :
Soit \displaystyle \sum_{n\geq 0} a_n z^n et \displaystyle \sum_{n \geq 0} b_n z^n deux séries entières de rayon de convergence R_a \text{ et } R_b respectivement. 
Si a_n \sim b_n, alors : R_a = R_b.
Proposition :
Soit \displaystyle \sum_{n\geq 0} a_n z^n et \displaystyle \sum_{n \geq 0} b_n z^n deux séries entières de rayon de convergence R_a \text{ et } R_b respectivement. 
S'il existe n_o \in \mathbb{N} tq : \forall n \geq n_o \, : \, |a_n| \leq |b_n|, alors : R_b \leq R_a.
Proposition :
Soit f une fonction rationnelle n'ayant pas de pôles entiers. Alors le rayon de convergence de \displaystyle \sum_{n\geq 0} f(n)z^n est égal à 1.

Remarque : 
L'utilisation de la règle de D'Alembert est pratique surtout pour les séries entières dites lacunaires : 
C'est-à-dire les séries de la forme \displaystyle \sum_{n \geq 0} b_n z^{\psi (n)} tq \psi une extractrice et : \displaystyle \sum_{n\geq 0} b_n z^{\psi (n)} = \displaystyle \sum_{n \geq 0} a_n z^n avec \left \lbrace \begin{array}{l} a_{\psi(n)} = b \\ a_k = 0 \text{ si } k \not \in \psi(\mathbb{N})} \end{array} \right . 

Exemple : 
    \displaystyle \sum n \frac{a^n}{2^n} z^{3n} avec a \in \mathbb{K}^* : 
Pour z \neq 0, on pose : b_n(z) = \displaystyle \frac{na^n}{2^n} z^{3n}
\forall n \geq 1 \, : \, \left| \displaystyle \frac{b_{n+1}(z)}{b_n(z)} \right| = \left| \displaystyle \frac{n+1}{n} \frac{a}{2} z^3 \right| = \displaystyle \frac{n+1}{n} \frac{|a|}{2} |z|^3 \xrightarrow[n\to+\infty]{} \displaystyle \frac{|a|}{2} |z|^3
      * Si \displaystyle \frac{|a|}{2} |z|^3 < 1 (ie |z| < \left(\displaystyle \frac{2}{|a|} \right)^{\frac{1}{3}}) : donc \displaystyle \sum n \displaystyle \frac{a^n}{2^n} z^{3n} converge absolument. 
      * Si \displaystyle \frac{|a|}{2} |z|^3 > 1 (ie |z| > \left(\displaystyle \frac{2}{|a|}\right)^{\frac{1}{3}}) : alors \displaystyle \sum n\frac{a^n}{2^n} z^{3n} diverge. 
On déduit que R = \left(\displaystyle \frac{2}{|a|}\right)^{\frac{1}{3}} 
Proposition :
Soit \displaystyle \sum_{n \geq 0} a_n z^n une série entière, soit R (resp. R_1 resp. R_2) le rayon de convergence de \displaystyle \sum_{n \geq 0} a_n z^n(resp. \displaystyle \sum_{n\geq 0} a_{2n} z^{2n} et \displasystyle \sum_{n \geq 0} a_{2n+1} z^{2n+1}). Alors R = \min(R_1  ,R_2)
De plus : \forall z \in \mathbb{K} \, , \, |z| < R \, : \, \displaystyle \sum_{n= 0}^{+\infty} a_{n} z^{n} = \displaystyle \sum_{n= 0}^{+\infty} a_{2n} z^{2n} + \displaystyle \sum_{n= 0}^{+\infty} a_{2n+1} z^{2n+1}.

Exemple : \displaystyle \sum_{n\geq 1} a_n z^n avec : a_n = n^{(-1)^n} : 
\displaystyle \sum a_{2n}z^{2n} = \displaystyle \sum 2n z^{2n}, donc R_1 = 1
\displaystyle \sum a_{2n+1}z^{2n+1} = \displaystyle \sum \frac{1}{2n+1}z^{2n}, donc R_2 = 1
On en déduit que \boxed{R = 1} . 
Proposition :
Soit \displaystyle \sum_{n \geq 0} a_n z^n une série entière de rayon de convergence R, soit f une fonction rationnelle n'ayant pas de pôles dans \mathbb{N}
Alors le rayon de convergence de \displaystyle \sum f(n) a_n z^n est égal à R.


2. Convergence ponctuelle

Théorème :
Soit \displaystyle \sum_{n\geq 0} a_n z^n une série entière de rayon de convergence R > 0
Alors \displaystyle \sum_{n\geq 0} a_n z^n converge normalement sur tout compact inclus dans D(0 , R)

Remarque : 
Il se peut qu'une série entière de rayon de convergence positif ne converge pas normalement sur le disque D(0 , R) .
Corollaire :
La somme d'une série entière de rayon de convergence positif est continue sur le disque D(0 , R) .



III. Opérations sur les séries entières

* Soit \displaystyle \sum_{n \geq 0} a_n z^n\displaystyle \sum_{n \geq 0 } b_n z^n deux séries entières de rayons de convergence R_a et R_b respectivement. 
* Soit \lambda \in \mathbb{C}
* On pose : S_n = a_n + b_nt_n = \lambda a_n et c_n = \displaystyle \sum_{k=0}^n a_k b_{n-k}
* On note R_S \, , \, R_t et R_c les rayons de convergence respectivement des séries entières : \displaystyle \sum S_n z^n\displaystyle \sum t_n z^n et \displaystyle \sum c_n z^n

i) 
* R_S \geq \min(R_a,R_b)
* \forall z \in \mathbb{K} \, : \, |z| \leq \min(R_a , R_b) \, \Longrightarrow \, \displaystyle \sum_{n=0}^{+\infty}(a_n + b_n)z^n = \displaystyle \sum_{n=0}^{+\infty} a_n z^n +  \displaystyle \sum_{n=0}^{+\infty} b_n z^n

ii) 
* R_t = \left \lbrace \begin{array}{l} +\infty  \text{ si } \lambda = 0 \\ R_a \text{ si } \lambda \neq 0 \\ \end{array} \right . 
* \forall z \in \mathbb{K} \, : \, |z| < R_a \, \Longrightarrow \, \displaystyle \sum_{n=0}^{+\infty} \lambda a_n z^n = \lambda \displaystyle \sum_{n=0}^{+\infty}  a_n z^n

iii) 
* R_c \geq \min(R_a,R_b)
* \forall z \in \mathbb{K} \, : \, |z| \leq \min(R_a,R_b) \, \Longrightarrow \, \displaystyle \sum_{n=0}^{+\infty} \left(\displaystyle \sum_{k=0}^n a_k b_{n-k} \right)z^n = \left(\displaystyle \sum_{n=0}^{+\infty} a_n z^n \right) \left(\displaystyle \sum_{n=0}^{+\infty} b_n z^n \right)


IV. Propriétés de la somme d'une série entière de la variable réelle

Ici, \displaystyle \sum_{n\geq 0} a_n x^n est une série entière de la variable réelle dont le rayon de convergence R est supposé positif et dont la somme est noté f
f est définie sur au moins ]-R , R[, on rappelle que f est continue sur cet intervalle. 
Théorème :
La série entière \displaystyle \sum_{n\geq 0} a_n \displaystyle \frac{x^{n+1}}{n+1} est de rayon de convergence R

De plus, on a : \forall x \in ]-R,R[ \, : \, \displaystyle \sum_{n=0}^{+\infty} a_n \displaystyle \frac{x^{n+1}}{n+1} = \displaystyle \int_0^{x} f(t) \text{d}t

Remarque : 
Soit p , q \in ]-R , R[ \, : \, \displaystyle \int_{p}^q f(t) \text{d}t = \displaystyle \int_{0}^q f(t) \text{d}t - \displaystyle \int_{0}^p f(t) \text{d}t
\displaystyle \int_{p}^q f(t) \text{d}t = \displaystyle \sum_{n=0}^{+\infty} a_n \displaystyle \frac{q^{n+1}}{n+1} - \displaystyle \sum_{n=0}^{+\infty} a_n \displaystyle \frac{p^{n+1}}{n+1} = \displaystyle \sum_{n=0}^{+\infty} a_n \displaystyle \frac{q^{n+1}-p^{n+1}}{n+1} = \displaystyle \sum_{n=0}^{+\infty} \displaystyle \int_{p}^q a_n t^n \text{d}t
Autrement dit : \displaystyle \int_p^q \left(\displaystyle \sum_{n=0}^{+\infty} a_n t^n \right) \text{d}t = \displaystyle \sum_{n=0}^{+\infty} \left(\displaystyle \int_p^q a_n t^n \text{d}t\right)
On dit qu'une série entière s'intègre terme à terme entre deux points quelconques de son intervalle de convergence . 
Théorème :
* Pour tout k \in \mathbb{N}, la série entière \displaystyle \sum_{n \geq k} a_n \displaystyle \frac{n!}{(n-k)!} x^{n-k} a pour rayon de convergence R
* f est de classe \mathfrak{C}^{\infty} sur ]-R , R[ et on a : \forall k \in \mathbb{N} \, , \, \forall x \in ]-R , R[  : f^{(k)} (x) = \displaystyle \sum_{n=k}^{+\infty} a_n \displaystyle \frac{n!}{(n-k)!} x^{n-k}.

Exemple : 
Soit f(x) = \displaystyle \sum_{n=2}^{+\infty} \displaystyle \frac{x^n}{n(n-1)}
On a : R = 1 ; f est donc définie et continue sur ]-1 , 1[
f(1) et f(-1) existent, donc : D_f = [-1 , 1]
\forall n \geq 2 \, , \, \forall x \in [-1,1] \, : \, \left| \displaystyle \frac{x^n}{n(n-1)} \right| \leq \displaystyle \frac{1}{n(n-1)} = \alpha_n
\displaystyle \sum \alpha_n converge, donc f est somme uniforme sur [-1 , 1] d'une série de fonctions continue sur [-1 , 1], alors f est continue sur [-1,1]
f est de classe \mathfrak{C}^\infty sur ]-1,1[ : 
f'(x) = \displaystyle \sum_{n=2}^\infty \frac{x^{n-1}}{n-1}   et   f''(x) = \displaystyle \sum_{n=2}^{+\infty} x^{n-2} = \displaystyle \sum_{n=0}^{+\infty} x^n = \displaystyle \frac{1}{1-x}
f'(x) = \displaystyle \int_{0}^x f''(t) \text{d}t (car f'(0) = 0). 
Donc : f'(x) = \displaystyle \int_{0}^x f''(t) \text{d}t = \displaystyle \int_{0}^x \displaystyle \frac{\text{d}t}{1-t} = -\ln(1-x)
et puisque f(0) = 0
f(x) = \displaystyle \int_{0}^x f'(t) \text{d}t = (1 - x) \ln(1 - x) + x \hspace{20pt} \forall x \in ]-1 , 1[
Donc : \boxed{f(x) = \displaystyle \sum_{n=2}^{+\infty} \displaystyle \frac{x^n}{n(n-1)} = (1 - x)\ln(1 - x) + x \hspace{20pt} \forall x \in ]-1 , 1[}

Remarque : 
On avait obtenu : \forall x \in ]-1 , 1[ \, : \, \displaystyle \sum_{n=2}^{+\infty} \displaystyle \frac{x^n}{n(n-1)} = (1-x)\ln(1-x) + x
Donc : f(-1) = \displaystyle \lim_{n\to-1^+} f(x) = 2\ln 2 -1, on en déduit : \displaystyle \sum_{n=2}^{+\infty} \displaystyle \frac{(-1)^n}{n(n-1)} = 2\ln2-1
Essayons de démontrer ce résultat directement en étudiant la somme partielle de la série : 
Soit N > 1 \, : 
\displaystyle \sum_{n=2}^{2N+1} \displaystyle \frac{(-1)^n}{n(n-1)} \\ = \displaystyle \sum_{k=1}^{N} \displaystyle \frac{1}{2k(2k-1)} - \displaystyle \sum_{k=1}^{N} \frac{1}{2k(2k+1)} \\ = \displaystyle \sum_{k=1}^{N} \left(\displaystyle \frac{1}{2k-1} - \displaystyle \frac{1}{2k}\right) - \displaystyle \sum_{k=1}^{N} \left(\displaystyle \frac{1}{2k} - \displaystyle \frac{1}{2k+1}\right)\\ = \displaystyle \sum_{k=1}^{N} \displaystyle \frac{1}{2k-1} + \displaystyle \sum_{k=1}^{N} \displaystyle \frac{1}{2k+1} - \displaystyle \sum_{k=1}^{N} \displaystyle \frac{1}{k} 
= \displaystyle \sum_{k=0}^{N-1} \displaystyle \frac{1}{2k+1} + \displaystyle \sum_{k=1}^{N} \displaystyle \frac{1}{2k+1} - \displaystyle \sum_{k=1}^{N} \displaystyle \frac{1}{k}\\ = 2\displaystyle \sum_{k=0}^{N-1} \displaystyle \frac{1}{2k+1} + 1 + \displaystyle \frac{1}{2N+1} - \displaystyle \sum_{k=1}^{N} \displaystyle \frac{1}{k}\\ = 2 \left(\displaystyle \sum_{k=1}^{N-1} \displaystyle \frac{1}{2k+1} + \displaystyle \sum_{k=1}^{N} \displaystyle \frac{1}{2k} - \displaystyle \sum_{k=1}^{N} \displaystyle \frac{1}{2k} \right) + 1 + \displaystyle \frac{1}{2N+1} - \displaystyle \sum_{k=1}^{N} \displaystyle \frac{1}{k} 
= 2 \left(\displaystyle \sum_{k=1}^{2N} \displaystyle \frac{1}{k}\right) - 2 \displaystyle \sum_{k=1}^{N} \displaystyle \frac{1}{k} - 2 + 1 + \displaystyle \frac{1}{2N + 1} \\ = 2 \left(\gamma_{2N} +\ln (2N)\right) - 2 \left(\gamma_N + \ln N\right) - 1 + \displaystyle \frac{1}{2N+1} \\ = 2 \gamma_{2N} - 2\gamma_{N} + 2\ln 2 - 1 + \displaystyle \frac{1}{2N+1} \xrightarrow[N \to +\infty]{} 2\ln2-1 
On conclut que la méthode des séries entières est plus performante que la méthode directe. 

Rappel : La constante d'Euler 
On pose : \gamma_n = \displaystyle \sum_{k=1}^n - \ln(n) 
Etudions la convergence de la suite (\gamma_n) en introduisant la série téléscopique \displaystyle \sum U_n tq : 
U_n = \gamma_n-\gamma_{n+1} \\ = - \displaystyle \frac{1}{1+n} + \ln(n+1)-\ln(n) \\ = - \displaystyle \frac{1}{n} \displaystyle \frac{1}{1 + \frac{1}{n}} + \ln \left(1+\frac{1}{n}\right) \\ = - \displaystyle \frac{1}{n} \left(1 - \displaystyle \frac{1}{n} + o \left(\displaystyle \frac{1}{n} \right) \right) + \left(\displaystyle \frac{1}{n} - \displaystyle \frac{1}{2n^2} + o \left(\displaystyle \frac{1}{n^2}\right)\right) \\ = \displaystyle \frac{1}{2n^2} + o \left(\displaystyle \frac{1}{n^2}\right) 
Donc : \displaystyle U_n \mathop{\sim}\limits_{n \to \infty} \frac{1}{n^2}
La suite étant réelle, U_n \geq 0 à partir d'un certain rang n_o (\displaystyle \sum U_n est à terme positif à partir du rang n_o) . 
Donc : \displaystyle \sum_{n\geq n_o} U_n converge, alors : \displaystyle \sum_{n\geq 1 } U_n converge. 
On en déduit que : (\gamma_n) converge. 
Définition :
La limite de la suite \left(\displaystyle \sum_{k=1}^{n} \displaystyle \frac{1}{k} - \ln(n) \right)_{n\geq 1} est appelée la constante d'Euler, on la note \gamma.


Remarque : 
On a :\displaystyle \sum_{k=1}^{n} \displaystyle \frac{1}{k} - \ln(n) - \gamma = o(1), d'où : \displaystyle \sum_{k=1}^{n} \frac{1}{k} = \ln(n) +\gamma + o(1) 


V. Développement en série entière (DSE)

1. Généralités

I est un intervalle de \mathbb{R} tq 0 \in I^o
Définition :
Soit f : I \longrightarrow \mathbb{C} et r > 0 tq :]-r , r[ \subset I
On dit que f est développable en série entière (DSE) sur ]-r , r[ ssi il existe une série entière \displaystyle \sum a_n x^n de rayon de convergence R \geq r tq : \forall x \in ]-r , r[ : f(x) = \displaystyle \sum_{n=0}^{+\infty} a_n x^n
De manière plus générale si x_0 \in I et si on dit que f est développable en série entière au voisinage de x_0 s'il existe r > 0tel que ]x_0 - r , x_0 + r[ \subset I et une série entière \displaystyle \sum a_n x^n de rayon de convergence R \geq r tels que \left(\forall x \in]x_0-r,x_0+r[ \right) \quad f(x) = \displaystyle \sum_{n=0}^{+\infty}a_n(x-x_0)^n

Définition :
Soit f : I \longrightarrow \mathbb{C}, on dit que f DSE au voisinage de 0 ssi il existe r > 0 tq : \left \lbrace \begin{array}{l} ]-r , r[ \subset I \\ f \text{ est DSE sur } ]-r,r[ \\ \end{array} 
On note : f \in DSE(0).


Exemples : 
* a \in \mathbb{C} \, , \, f : x \longrightarrow e^x 
La fonction f est DSE sur \mathbb{R} avec : \boxed{e^{ax} = \displaystyle \sum_{n=0}^{+\infty} \displaystyle \frac{a^n}{n!} x^n }
* \begin{array}{rccl} f : & ]-\infty , 1[ & \longrightarrow & \mathbb{R} \\  & x & \mapsto & \displaystyle \frac{1}{1-x} \\ \end{array} 
La fonction f est DSE sur ]-1 , 1[ avec : \boxed{\displaystyle \frac{1}{1-x} = \displaystyle \sum_{n=0}^{+\infty} x^n}
* Soit  a \in \mathbb{K} \backslash \lbrace 0 \rbrace f : x \mapsto \displaystyle \frac{1}{a-x} : 
f est définie sur ]-|a| , |a|[
Sur cet intervalle, f est DSE. 
En effet, f(x) = \displaystyle \frac{1}{a} \left( \displaystyle  \frac{1}{1 - \displaystyle \frac{x}{a}} \right) 
Or : \left| \displaystyle  \frac{x}{a} \right| = \displaystyle \frac{|x|}{|a|} < 1 
D'où : f(x) = \displaystyle \frac{1}{a} \displaystyle \sum_{n=0}^{+\infty} \left(\frac{x}{a} \right)^n, donc : \boxed{\displaystyle \frac{1}{a-x} = \displaystyle \sum_{n=0}^{+\infty} \displaystyle \frac{x^n}{a^{n+1}}}
* Soit f : \mathbb{R} \longrightarrow \mathbb{C} polynômiale de degré p : 
Alors : f(x) = a_0 + a_1x + \cdots + a_px^p avec a_i \in\mathbb{C} \, \forall i \in \lbrace 1,\cdots, p \rbrace et a_p \neq 0
En posant a_n = 0 \, \forall n > p
\displaystyle \sum_{n\geq 0} a_n x^n converge pour tout x \in \mathbb{R} de somme f(x)
Toute fonction polynômiale est DSE sur \mathbb{R} et elle est son propre développement. 
Théorème :
Soit f : I \longrightarrow \mathbb{C} DSE sur ]-r , r[ \subset I avec f(x) = \displaystyle \sum_{n=0}^{+\infty} a_n x^n. Alors : 
* f est de classe \mathfrak{C}^{\infty} sur ]-r , r[
* \forall n \in \mathbb{N} \, : \, a_n = \displaystyle \frac{f^{(n)}(0)}{n!}.


Exemple : 
* \begin{array}{rccl} f : & \mathbb{R} & \longrightarrow & \mathbb{R} \\  & x & \mapsto & \left \lbrace \begin{array}{l} \displaystyle \frac{e^x-1}{x} \text{ si } x \neq 0 \\ 1 \text{ si } x = 0 \\ \end{array} \right. \\ \end{array} 
Soit x \in \mathbb{R} ; on a : e^x = \displaystyle \sum_{n=0}^{+\infty} \displaystyle \frac{x^n}{n!} 
D'où : e^x - 1 = \displaystyle \sum_{n=1}^{+\infty} \displaystyle \frac{x^n}{n!}
Si x \neq 0 \, : \, \displaystyle \frac{e^x-1}{x} = \displaystyle \sum_{n=1}^{+\infty} \displaystyle \frac{x^{n-1}}{n!} = \displaystyle \sum_{n=0}^{+\infty} \displaystyle \frac{x^{n}}{(n+1)!} 
C'est-à-dire : f(x) = \displaystyle \sum_{n=0}^{+\infty} \displaystyle \frac{x^n}{(n+1)^!}
Et cela reste vrai pour x = 0; donc \forall x \in \mathbb{R} \, : \, f(x) = \displaystyle \sum_{n=0}^{+\infty} \displaystyle \frac{x^n}{(n+1)^!}
On conclut que f est DSE sur \mathbb{R} donc \mathfrak{C}^{\infty} sur \mathbb{R}
Corollaire :
Soit f : I \longrightarrow \mathbb{C} et r > 0 tq : ]-r , r[ \subset I
Si f est DSE sur ]-r , r[ avec f(x) = \displaystyle \sum_{n=0}^{+\infty} a_n x^n. Alors (a_n)_n est unique.


Exemple : 
Soit \begin{array}{rccl} f: & ]-1,1[ & \longrightarrow & \mathbb{R} \\  & x & \mapsto & \displaystyle \frac{1}{(1-x^2)(1+x)} \\ \end{array} 
* Méthode 1 : 
Pour |x| < 1 \, : \, \displaystyle \frac{1}{1-x^2} = \displaystyle \sum_{n=0}^{+\infty} x^{2n}   et   \displaystyle \frac{1}{1+x} =\displaystyle \sum_{n=0}^{+\infty} (-1)^n x^n
D'où, pour |x| < 1 \, : \, f(x) = \left(\displaystyle \sum_{n=0}^{+\infty} x^{2n}\right) \left(\displaystyle \sum_{n=0}^{+\infty} (-1)^n x^n \right)  = \left(\displaystyle \sum_{n=0}^{+\infty}a_n x^{n}\right) \left(\displaystyle \sum_{n=0}^{+\infty} (-1)^n x^n \right) avec : \left \lbrace \begin{array}{l} a_{2n} = 1 \\ a_{2n+1} = 0 \\ \end{array} \right. 
D'où : f(x) = \displaystyle \sum_{n=0}^{+\infty} \left(\displaystyle \sum_{k=0}^{n} a_k (-1)^{n-k} \right) x^n = \displaystyle \sum_{n=0}^{+\infty} \left(\displaystyle \sum_{k=0}^{E(\frac{n}{2})} a_{2k} (-1)^{n-2k} \right) x^n, pour |x| < 1 
Donc la méthode 1) nous fournit : \boxed{f(x) = \displaystyle \sum_{n=0}^{+\infty} (-1)^n \left(E \left(\displaystyle \frac{n}{2} \right) + 1 \right) x^n} pour |x| < 1

* Méthode 2 : 
On a : f(x) = \displaystyle \frac{1}{(1-x)(1+x)^2} et pour |x| < 1 : \displaystyle \frac{1}{1-x} = \displaystyle \sum_{n=0}^{+\infty} x^n
Or, \displaystyle \frac{1}{(1+x)^2} = \left(\frac{-1}{x+1}\right)' = -\left(\displaystyle \sum_{n=0}^{+\infty} (-1)^n x^n \right)' = -\displaystyle \sum_{n=1}^{+\infty} (-1)^{n} n x^{n-1} =\displaystyle \sum_{n=1}^{+\infty} (-1)^{n+1} n x^{n-1}
On obtient : \displaystyle \frac{1}{(1+x)^2} = \displaystyle \sum_{n=0}^{+\infty} (-1)^{n} (n+1) x^{n}
f(x) = \left( \displaystyle \sum_{n=0}^{+\infty}x^n \right) \left(\displaystyle \sum_{n=0}^{+\infty} (-1)^{n} (n+1) x^{n} \right) 
Donc la méthode 2) nous fournit : \boxed{f(x) = \displaystyle \sum_{n=0}^{+\infty} \left(\displaystyle \sum_{n=0}^{n} (-1)^{k} (k+1) \right) x^{n}}

* Méthode 3 : 
Décomposons f en fractions rationnelles : f(x) = \displaystyle \frac{a}{1-x} + \displaystyle \frac{b}{1+x} + \displaystyle \frac{c}{(1+x)^2} 
On obtient par calcul : a = \displaystyle \frac{1}{4} \, , \, c = \displaystyle \frac{1}{2} \text{ et } b = \displaystyle \frac{1}{4} \left(f(0)  =  1  = a + b + c \right) 
f(x) = \displaystyle \frac{1}{4(1-x)} + \displaystyle \frac{1}{4(1+x)} + \displaystyle \frac{1}{2(1+x)^2} \\ = \frac{1}{4} \displaystyle \sum_{n=0}^{+\infty} x^n + \displaystyle  \frac{1}{4} \displaystyle \sum_{n=0}^{+\infty} (-1)^n x^n + \displaystyle  \frac{1}{2} \displaystyle \sum_{n=0}^{+\infty} (-1)^n (n+1)x^n \\ = \displaystyle \sum_{n=0}^{+\infty} \displaystyle \frac{1+(-1)^n+2(-1)^n(n+1)}{4} x^n = \displaystyle \sum_{n=0}^{+\infty} \displaystyle  \frac{1+(-1)^n(2n+3)}{4} x^n
La méthode nous fournit : \boxed{f(x) = \displaystyle \sum_{n=0}^{+\infty} \displaystyle \frac{1+(-1)^n(2n+3)}{4} x^n} 
Par unicité du DSE sur ]-1 , 1[ , on a : \boxed{(-1)^n \left(E \left(\displaystyle \frac{n}{2} \right) + 1\right) = \displaystyle \sum_{k=0}^n (-1)^k (k+1) = \displaystyle \frac{1+(-1)^n(2n+3)}{4}} 
Définiton :
Soit f : I \longrightarrow \mathbb{C} de classe \mathfrak{C}^{\infty}
La série entière \displaystyle \sum_{n\geq 0} \displaystyle \frac{f^{(n)} (0) }{n!} x^n est appelée la série de Taylor de f.



2. Techniques de calcul de DSE au voisinage de 0

Il existe plusieurs méthodes de calcul d'un DSE d'une fonction, nous citerons ici les plus utilisées, mais cela ne veut pas dire qu'elles sont les seules, on peut très bien avoir affaire à d'autres méthodes. 

Notations et vocabulaire : 
* Si f(x) = \displaystyle \sum_{n=0}^{+\infty} a_n x^n (avec |x| < r) est un DSE sur ]-r , r[ alors le rayon de convergence de la série entière \displaystyle \sum_{n\geq 0} a_n x^n est appelé le rayon de convergence du DSE de f
* Le plus grand intervalle ouvert ]-r , r[ sur lequel ce développement est valable s'appelle le domaine de validité du DSE de f, on le note D_v

Remarque : 
D_v \neq D_f en général. (Avec D_v est le domaine de validité d'un DSE de f). 

Contre exemple : 
\begin{array}{rccl} f: & \mathbb{R} & \longrightarrow& \mathbb{R} \\   & x & \mapsto & \displaystyle \frac{1}{1+x^2} \\ \end{array}       on a : D_v = ]-1 , 1[ et D_f = \mathbb{R}


Technique 1 : Utilisation des opérations sur les séries entières

Supposons : f(x) = \displaystyle \sum_{n=0}^{+\infty} a_n x^n|x| < r_1   et   g(x) = \displaystyle \sum_{n=0}^{+\infty} b_n x^n|x| \leq r_2 
En posant r = \min(r_1,r_2), on a : 
* \forall \lambda, \eta \in \mathbb{C} : \lambda f(x) + \eta g(x) = \displaystyle \sum_{n=0}^{+\infty} \left(\lambda a_n + \eta b_n \right) x^n ; R \geq r pour |x| < r
* f \times g(x) = \displaystyle \sum_{n=0}^{+\infty} \left(\displaystyle \sum_{k=0}^{n} a_k b_{n-k} \right) x^n ; R \geq r pour |x| < r
Donc toute combinaison linéaire (resp. produit) de deux fonctions DSE(0) est une fonction DSE(0). 

Exemples : 
* \forall x \in \mathbb{R} \, : \, x \longrightarrow ch(x) \text{ et } x \longrightarrow sh(x) : 
On a : \forall x \in \mathbb{R} \, : \, ch(x) = \frac{e^x+e^{-x}}{2} 
De plus : e^x = \displaystyle \sum_{n=0}^{+\infty} \displaystyle \frac{x^n}{n!} donc : e^{-x} = \displaystyle \sum_{n=0}^{+\infty} \displaystyle \frac{(-1)^n}{n!} x^n 
Alors : ch x = \displaystyle \sum_{n=0}^{+\infty} \displaystyle \frac{1+(-1)^n}{2} \frac{x^n}{n!} \, \, \forall x \in \mathbb{R} 
Donc : \boxed{ch x = \displaystyle \sum_{n=0}^{+\infty} \displaystyle \frac{x^{2n}}{(2n)!} \, , \, x \in \mathbb{R}} 
De même : \boxed{sh x = \displaystyle \sum_{n=0}^{+\infty} \displaystyle \frac{x^{2n+1}}{(2n+1)!} \, , \, x \in \mathbb{R}} 
* \forall x \in \mathbb{R} \, , \, x \longrightarrow \cos x  \text{ et }  x \longrightarrow \sin x : 
On a : \forall x \in \mathbb{R} \, \, \cos(x) = \displaystyle \frac{e^{ix}+e^{-ix}}{2} : 
On a : e^{ix} = \displaystyle \sum_{n=0}^{+\infty} \displaystyle \frac{i^n x^n}{n!} et e^{-ix} = \displaystyle \sum_{n=0}^{+\infty} \displaystyle \frac{(-i)^n x^n}{n!} 
Alors : \cos(x) = \displaystyle \sum_{n=0}^{+\infty} \displaystyle \frac{i^n +(-i)^n}{2} \displaystyle \frac{x^n}{n!} = \displaystyle \sum_{n=0}^{+\infty}i^{2n} \displaystyle \frac{x^{2n}}{2n!}
On conclut : \boxed{\cos(x) = \displaystyle \sum_{n=0}^{+\infty} (-1)^n \displaystyle \frac{x^{2n}}{(2n)!} \, , \, x \in \mathbb{R}} 
De même : \boxed{ \sin(x) = \displaystyle \sum_{n=0}^{+\infty} (-1)^n \displaystyle \frac{x^{2n+1}}{(2n+1)!} \, , \,  x \in \mathbb{R}} 


Technique 2 : Méthode de la dérivation et intégration

Soit f : I \longrightarrow \mathbb{C} dérivable tq f' admet un DSE sur ]-r , r[ \subset I donné par : f'(x) = \displaystyle \sum_{n=0}^{+\infty} a_n x^n
Alors : \displaystyle \int_{0}^x f'(t)dt = \displaystyle \sum_{n=0}^{+\infty} \displaystyle \frac{a_n}{n+1} x^{n+1} \, , \, \forall x \in ]-r , r[
D'où, \forall x \in ]-r , r[ \, : \, f(x) = f(0) + \displaystyle \sum_{n=0}^{+\infty} \displaystyle \frac{a_n}{n+1}x^{n+1}

Exemples : 
* \begin{array}{rccl} f : & ]-1 , +\infty[ & \longrightarrow & \mathbb{R} \\   & x & \mapsto & \ln(1+x) \\ \end{array} 
f est dérivable et f'(x) = \displaystyle \frac{1}{1+x} \, \forall x \in ]-1 , +\infty[
Or \displaystyle \frac{1}{1+x} = \displaystyle \sum_{n=0}^{+\infty} (-1)^n x^n pour tout x tq : |x| < 1
D'où : f(x) = f(0) + \displaystyle \sum_{n=0}^{+\infty} \displaystyle \frac{(-1)^n}{n+1} x^{n+1} avec |x| < 1
C'est-à-dire : \boxed{\ln(1+x) = \displaystyle \sum_{n=1}^{+\infty} \displaystyle \frac{(-1)^{n-1} x^n}{n} \, , \, |x| < 1}
* \begin{array}{rccl} f : & \mathbb{R} & \longrightarrow& \mathbb{R} \\   & x & \mapsto & \arctan(x)\\ \end{array} 
f est dérivable sur \mathbb{R}   et   f'(x) = \displaystyle \frac{1}{1+x^2}
On a : \displaystyle \frac{1}{1+x^2} = \displaystyle \sum_{n=0}^{+\infty} (-1)^n x^{2n} avec |x| < 1
D'où : f(x) = f(0) + \displaystyle \sum_{n=0}^{+\infty} \displaystyle \frac{(-1)^n}{2n+1}  x^{2n+1} 
Donc : \boxed{\arctan(x) = \displaystyle \sum_{n=0}^{+\infty} \displaystyle \frac{(-1)^n}{2n+1} x^{2n+1} \, , \, |x| < 1}


Technique 3 : Utilisation d'une équation differentielle linéaire

On considère l'équation différentielle linéaire d'ordre n : 
(E) : y^{(n)} + a_{n-1}(x) y^{(n-1)} +\cdots + a_0(x) y = b(x) où : a_0,\cdots , a_n , b \in\mathfrak{C}^o(I,\mathbb{C})
Etant donné n_0 \in I et c_0 , \cdots , c_{n-1} \in \mathbb{C}, on admet qu'il existe une unique solution \psi de (E) sur I tq : 
\left. \begin{array}{rcl} \psi(x_0) & = & c_0 \\ \psi '(x_0) & = & c_1\\ \vdots & & \\ \psi^{(n-1)}(x_0) & = & c_{n-1} \\ \end{array} \right \rbrace  

Soit f : I \longrightarrow \mathbb{C} de classe \mathfrak{C}^{\infty}
Si on détermine toutes les solutions de (E), DSE(0) et si l'une d'elle, \psi, vérifie : 
\left. \begin{array}{rcl} \psi(0) & = & f(0) \\ \psi '(0) & = & f'(0) \\ \vdots & & \\ \psi^{(n-1)}(0) & = & f^{(n-1)}(0) \\ \end{array} \right \rbrace  

Alors f = \psi, d'où f est DSE(0). 
Dans la pratique, on se contentera du cas où les fonctions a_0 , \cdots , a_{n-1}, b sont rationnelles. 

Exemple : 
*  \begin{array}{rccl} f : & ]-1 , +\infty[ & \longrightarrow& \mathbb{C} \\  & x & \mapsto & (1+x)^a \\ \end{array} (a \in \mathbb{C} \backslash \mathbb{Z} ) 
f est dérivable et f'(x) = a(1 + x)^{a - 1}
(1 + x)f'(x) = a(1 + x)^a = af(x) 
f est donc solution sur ]-1 , +\infty[ de l'équation différentielle (e) : (1+x)y' - ay = 0 \Longleftrightarrow y' - \displaystyle \frac{a}{x+1} y = 0
Soit r \in ]0 , 1] , soit \psi : ]-r , r[ \longrightarrow \mathbb{C} de classe \mathfrak{C}^{\infty} et DSE sur ]-r , r[
Posons : \psi(x) = \displaystyle \sum_{n=0}^{+\infty} a_n x^{n} avec |x| < r
\psi '(x) = \displaystyle \sum_{n=1}^{+\infty} n a_nx^{n-1}
On a : 
\psi est solution de (e) sur ]-r , r[ \Longleftrightarrow \, \forall x \in ]-r , r[ \, : \, (1 + x) \displaystyle \sum_{n=1}^{+\infty} n a_n x^{n-1} - a \displaystyle \sum_{n=0}^{+\infty} a_n x^n = 0 
\Longleftrightarrow \, \forall x \in ]-r , r[ \, : \, \displaystyle \sum_{n=1}^{+\infty} na_n x^{n-1} + \displaystyle \sum_{n=1}^{+\infty} n a_n x^n - a \displaystyle \sum_{n=0}^{+\infty} a_n x^n = 0 \\ \Longleftrightarrow \, \forall x \in ]-r , r[ \, : \, \displaystyle \sum_{n=0}^{+\infty} (n+1)a_{n+1} x^n + \displaystyle \sum_{n=1}^{+\infty} n a_n x^n - a\displaystyle \sum_{n=0}^{+\infty} a_n x^n = 0 \\ \Longleftrightarrow \forall x \in ]-r , r[ \, : \, \displaystyle \sum_{n=0}^{+\infty} \left[(n + 1) a_{n+1} - (a - n)a_n \right] x^n = 0
Par unicité du DSE sur ]-r , r[ de la fonction nulle. 
\psi est solution de (e) sur ]-r , r[ \, \Longleftrightarrow \, \forall n \in \mathbb{N} \, : \, a_{n+1} = \displaystyle \frac{a-n}{n+1} a_n \, \, (*) 

Remarque : 
Formellement : \left| \displaystyle \frac{a_{n+1} x^{n+1}}{a_n x^n} \right| = \displaystyle \frac{|a-n|}{n+1}|x| \longrightarrow_{n\to \infty} |x|. On pouvait donc prendre r = 1
(*) \, \Longleftrightarrow \, \forall n \in \mathbb{N}^* \, : \, a_n = \displaystyle \frac{a(a-1) \cdots (a-n+1)}{n!} a_0 = {{a}\choose {n}} a_0 
\psi est solution de (e) sur ]-r , r[ \, \Longleftrightarrow \, \forall x\in]-r , r[ \, : \, \psi(x) = a_0 \displaystyle \sum_{n=0}^{+\infty} {a\choose n} x^n 
Or, \psi(0) = f(0) \, \Longleftrightarrow \, a_0 = 1
On a donc trouvé le DSE de f sur ]-r , r[ et qui est : \boxed{(1+x)^a = \displaystyle \sum_{n=0}^{+\infty} {a\choose n} x^n}



Suites numériques

I. L'ensemble des suites réelles

1. Définition d'une suite

On appelle suite réelle toute application d'une partie de \mathbb{N} à valeurs dans \mathbb{R} et on note (U_{n})_{n} ou (V_{n})_{n} en général.
Vocabulaire : 
L'ensemble des suites réelles sera noté par \mathbb{R}^{\mathbb{N}

2. Opérations algébriques 
a) Egalité

Soient (U_{n}) et (V_{n}) \; \in \mathbb{R}^{\mathbb{N}}
On dit que les suites sont égales et on note : (U_{n}) = (V_{n}) ssi : (\forall n \in \mathbb{N}) : U_{n}=V_{n}

Exemple : 
\forall n \in \mathbb{N} : U_{n}= cos(n \Pi) et V_{n} = (-1)^{n} 
On a \forall n \in \mathbb{N} : cos(n\Pi) = (-1)^{n} donc \forall n \in \mathbb{N} : U_{n} = V_{n} alors : (U_{n})=(V_n)

b) Addition

Soient (U_n) et (V_n) \in \mathbb{R}^{\mathbb{N}}
La suite (W_n) \in \mathbb{R}^{\mathbb{N}} définie par : \forall n : \; W_n = U_n +V_n est appelée la suite somme de (U_n) et (V_n) et on note : (W_n) = (U_n+V_n).
Exemple : 
\forall n \in \mathbb{N} \; : \; U_n = n^2+1 \text{ et } V_n = 2n 
On a donc : W_n = U_n + V_n = n^2+1+2n = (n+1)^2 alors : W_n = (n+1)^2

c) Produit

Soient (U_n) et (V_n) \in \mathbb{R}^{\mathbb{N}}, la suite (W_n) \in \mathbb{R}^{\mathbb{N}} définie par : \forall n : \; W_n = U_n . V_n est appelée la suite produit de (U_n) et (V_n) et on note : (W_n) = (U_n.V_n)
Exemple : 
\forall n \in \mathbb{N} \; : \; U_n = (n+1)^2 \text{ et } V_n = \frac{1}{\sqrt{n+1}} 
On a : W_n = U_n.V_n = (n+1)^2.\frac{1}{\sqrt{n+1}}=(n+1).\sqrt{n+1} alors : W_n = (n+1).\sqrt{n+1}

d) Produit par un scalaire

Soit (U_n) \in \mathbb{R}^{\mathbb{N}} et soit \lambda \in \mathbb{R}, la suite (W_n) définie par : \forall n \in \mathbb{N} \; : \; W_n= \lambda U_n sera noté : (\lambda U_n)

3. Suite minorée - majorée - bornée

Définition :

Soit (U_n) \in \mathb{R}^{\mathbb{N}}
On dit que (U_n) est majorée (respectivement minorée) ssi : 
\exists M \in \mathbb{R}, \; \forall n \in \mathbb{N} \; : \; U_n \leq M (respectivement M \leq U_n). 
On dit que (U_n) est bornée si elle est à la fois majorée et minorée.
Exemple : 
\forall n \in \mathbb{N} \; : \; U_n = \sin(n^4+3n^3+5)
Puisque \forall n \in \mathbb{N} \; : \; -1 \leq \sin(n^4+3n^3+5) \leq 1 (Propriété de la fonction \sin
Alors \forall n \in \mathbb{N} \; : \; -1 \leq U_n \leq 1, c'est-à-dire que (U_n) est bornée .
Proposition :

Soit (U_n) \in \mathb{R}^{\mathbb{N}}(U_n) est bornée ssi : 
\exists M \in \mathbb{R}^{+} \; / \; \forall n \in \mathbb{N} ) \; : \; |U_n| \leq M.

4. Suite croissante - décroissante - monotone

Définition :

Soit (U_n) \in \mathb{R}^{\mathbb{N}}, on dit que : 
(U_n) est croissante (resp. strictement croissante) ssi : \forall n \in \mathbb{N} \; : \; U_n \leq U_{n+1} (respU_n < U_{n+1}
(U_n) est décroissante (resp. strictement décroissante) ssi : \forall n \in \mathbb{N} \; : \; U_n \geq U_{n+1} (resp U_n >U_{n+1}
(U_n) est monotone ssi elle est croissante ou décroissante. 
(U_n) est strictement monotone ssi elle est strictement croissante ou strictement décroissante.
Exemple : 
Soit (U_n) \in \mathb{R}^{\mathbb{N}} \; / \; \forall n \in \mathbb{N}) \; : \; U_n = \Bigsum_{k=0}^n~\frac{1}{k!} 
Soit n \in \mathbb{N} \; : \; U_{n+1} - U_n = \Bigsum_{k=0}^{n+1}~\frac{1}{k!} -\Bigsum_{k=0}^n~\frac{1}{k!} =\frac{1}{(n+1)!}>0 
Donc (U_n) est croissante. (Même strictement croissante) 

II. Notions de suites convergentes

1. Définition et propriétés de la convergence

Définitions :

Soit (U_n) \in \mathb{R}^{\mathbb{N}} et soit l \in \mathbb{R}
On dit que (U_n) converge vers l ssi : 
(\forall \epsilon >0 ), ( \exists N \in \mathbb{N}) \; , \; (\forall n \in  \mathbb{N}) \; : \; n \geq N \Rightarrow |U_n-l| \leq \epsilon 
Si une suite ne converge pas, on dit alors qu'elle diverge.
Vocabulaire - Notation : 
l est appelé limite de la suite (U_n) et on note : \displaystyle \lim_{n \to +\infty} U_n = l ou lim_{ } U_n = l.
Théorème :

Si la suite (U_n) converge vers une limite l, cette limite l est unique.
Proposition :

Toute suite convergente est bornée.
Remarque : 
La réciproque est fausse en général. 

Exemple : 
Soit (U_n) \in \mathb{R}^{\mathbb{N}} \; / \forall n \in \mathbb{N} \; : \; U_n = (-1)^n 
(U_n) est bornée mais pas convergente. 

2. Opérations sur les limites

Théorème :

Soient (x_n) et (y_n) deux suites de \mathbb{R}^{\mathbb{N}}
Soient l et l' de \mathbb{R}, tq : (x_n) converge vers l et (y_n) converge vers l', alors on a : 
\forall \lambda \in \mathbb{R}, la suite (x_n + \lambda y_n) est convergente et on a : \displaystyle \lim   (x_n + \lambda y_n) = l + \lambda l'= \displaystyle \lim x_n + \lambda . \displaystyle \lim y_n
La suite (x_n.y_n) est convergente et on a : \displaystyle \lim (x_n.y_n) = l \times l' = lim {x_n} \times lim {y_n}
Si l' \not = 0, alors \exists N \in \mathbb{N} \; / \; \forall n \geq N \; : \; y_n \not = 0, de plus, dans ce cas, la suite \left( \frac{x_n}{y_n}\right) est convergente et on a : lim \left(\frac{x_n}{y_n}\right) = \frac{l}{l'} = \frac{\displaystyle \lim x_n }{\displaystyle \lim y_n }.
Théorème :

Soient (x_n),(y_n) et (z_n) trois suites de \mathbb{R}^{\mathbb{N}} tq : \forall n \in \mathbb{N} \; y_n \leq x_n \leq z_n
Si \displaystyle \lim y_n = \displaystyle \lim z_n = l \in \mathbb{R}, alors (x_n) converge aussi vers l.


3. Convergence des suites monotones

Théorème fondamental :

Soit (U_n) \in \mathb{R}^{\mathbb{N}} une suite croissante, (U_n) converge ssi elle est majorée, et on a, dans ce cas : \displaystyle \lim \: U_n = Sup(U_n) 
Soit (U_n) \in \mathb{R}^{\mathbb{N}} une suite decroissante, (U_n) converge ssi elle est minorée, et on a, dans ce cas : \displaystyle \lim \: U_n = Inf(U_n)
Corollaire :

Toute suite croissante négative est convergente. 
Toute suite décroissante positive est convergente.
Définition :

Soit (U_n) \in \mathb{R}^{\mathbb{N}}
On dit que (U_n) tend vers + \infty (resp -\infty) ssi : (\forall A >0), (\exists N \in \mathbb{N}), (\forall n \in \mathbb{N}),n \geq N \Rightarrow U_n \geq A (resp : U_n \leq A ), et on note : \displaystyle \lim \: U_n= +\infty (resp -\infty).
Proposition :

Soit (U_n) \in \mathb{R}^{\mathbb{N}}
Si (U_n) est croissante et non majorée alors : \displaystyle \lim \: U_n = +\infty
Si (U_n) est décroissante et non minorée alors : \displaystyle \lim \: U_n = -\infty.
Proposition :

Soient (U_n) et (V_n) \in \mathbb{R}^{\mathbb{N}} tq : \forall n \in \mathbb{N} \; : \; U_n \geq V_n 
Si \displaystyle \lim \: U_n = -\infty alors \displaystyle \lim \: V_n = -\infty
Si \displaystyle \lim \: V_n = +\infty alors \displaystyle \lim \: U_n = +\infty.


III. Suites extraites - Suites adjacentes

1. Suites extraites

Définition :

Soient (U_n) et (V_n) \in \mathbb{R}^{\mathbb{N}}, on dit que (V_n) est une suite extraite de (U_n) ssi : \exist\begin{array}{rcccl} \psi&:&\mathbb{N}&\to& \mathbb{N}\\ \end{array} strictement croissante tq : \forall n \in \mathbb{N} \; : \; V_n = U_{\psi(n)} et on appelle \psiune extractrice.
Exemple : 
Soit (U_n) \in \mathb{R}^{\mathbb{N}} tq : U_n = (-1)^n et soient : \begin{array}{rcccl} \psi_1&:&\mathbb{N}&\to& \mathbb{N}\\ & &n &\mapsto &2n\end{array} et \begin{array}{rcccl} \psi_2&:&\mathbb{N}&\to& \mathbb{N}\\ & &n &\mapsto &2n+1\end{array} 
(V_n) et (W_n) sont extraites de (U_n) tq : V_n = U_{\psi_1 (n)} = U_{2n}=(-1)^{2n} = 1 et W_n = U_{\psi_2 (n)} = U_{2n+1}=(-1)^{2n+1} = -1
Proposition :

Soit \psiune extractrice alors : \forall n \in \mathbb{N} : \psi(n) \geq n
Proposition :

Toute suite extraite d'une suite bornée est bornée. 
Toute suite extraite d'une suite convergente est convergente et converge vers la même limite. 
Alors si on trouve que deux suites extraites ne convergent pas vers la même limite, alors la suite "mère" n'est pas convergente (diverge).
Exemple : dans l'exemple précédent, 
\displaystyle \lim \: V_n = 1 et \displaystyle \lim \: W_n = -1, alors (V_n) et (W_n) ne convergent pas vers la même limite (1 \not = -1), ce qui montre que (U_n) diverge.
Théorème de Bolzano - Weierstrass réel :

De toute suite réelle bornée on peut extraire une suite convergente.


2. Suites adjacentes

Définition :

Soient (U_n) et (V_n) \in \mathbb{R}^{\mathbb{N}}, on dit qu'elles sont adjacentes si l'une est croissante et l'autre est décroissante et \displaystyle \lim \: (U_n - V_n) = 0
Proposition :

Soient (U_n) et (V_n) deux suites adjacentes tq : (U_n) croissante et (V_n) décroissante, alors : 
\forall n \in \mathbb{N} \; : \; U_n \leq V_n.
Théorème :

Deux suites adjacentes sont convergentes et convergent vers la même limite.


IV. Suite de Segments Emboités

Définitions :

Un segment est tout intervalle fermé et borné de la forme [a,b] (a<b). 
Soit (I_n) une suite de segments, on dit que cette suite est emboitée ssi : (\forall n \in \mathbb{N} ) \; : \; I_{n+1} \subset I_n.
Exemple : 
Soit n \in \mathbb{N}^*, on note : I_n = [0,\frac{1}{n}](I_n) est une suite de segments emboités car \forall n \in \mathbb{N}^* \; [0,\frac{1}{n+1}] \subset [0, \frac{1}{n}].
Proposition :

Soit (I_n) = ([a_n,b_n]) une suite de segments, on a : (I_n) est emboité \Leftrightarrow ( (a_n) croissante et (b_n) décroissante).
Théorème des segments emboités :

Soit (I_n) = ([a_n,b_n]) une suite de segments emboités tq \displaystyle \lim(b_n - a_n) = 0, alors il existe un c \in \mathbb{R} tq : \bigcap_{n \in \mathbb{N}} [a_n,b_n]  =\lbrace c\rbrace


V. Suites récurrentes

1. Suite arithmético-géométrique

Définition :

Soit (U_n) \in \mathb{R}^{\mathbb{N}}, on dit que (U_n) est arithmético-géométrique ssi : \exists (a,b) \in \mathbb{R}^* -\lbrace 1\rbrace  \times \mathbb{R}^* tq : 
    U_0 \in \mathbb{R} \\ \forall n \in \mathbb{N} \; : \; U_{n+1}=aU_n+b
Remarque : 
Si a = 1 : (U_n) est arithmétique, et si b = 0 : (U_n) est géométrique.
Proposition :

(U_n) converge ssi : |a| < 1, et dans ce cas : \displaystyle \lim \: U_n = \frac{b}{1-a}


2. Suites récurrentes linéaires du 2e ordre

Définition :

Soit (a,b) \in \mathbb{R}^2, la suite (U_n) definie par : 
(U_0 ,U_1) \in \mathbb{R}^2 \\ \forall n \in \mathbb{N}  \; : \; U_{n+2} + aU_{n+1} + bU_n = 0 
est appelée suite recurrente linéaire du 2e ordre, et l'équation x^2 + ax + b = 0 \; (x \in \mathbb{C} ) est appelé équation caractéristique qu'il faut resoudre pour trouver (U_n) en fonction de n.
Théorème :

Soit x^2+ax+b=0 l'équation caractéristique et \triangle = a^2-4b.
  • Si \triangle > 0 : l'équation admet 2 racines réelles r_1 et r_2 tq : U_n = \alpha r_1^n+\beta r_2^n avec (\alpha, \beta) \in \mathbb{R}^2.
  • Si \triangle = 0 : l'équation admet une racine double réelle r tq : U_n = (\alpha n+\beta)r^n avec (\alpha, \beta) \in \mathbb{R}^2.
  • Si \triangle < 0 : l'équation admet deux racines conjuguées : r_1= r e^{i\theta} et r_2= r e^{-i\theta} tq : U_n=r^n(\alpha cos(n\theta)+\beta sin(n\theta)) avec (\alpha, \beta) \in \mathbb{R}^2.


3. Suite recurrente : cas général

Définition :

Soit I un intervalle de \mathbb{R} et soit \begin{array}{rcccl} f&:&I&\to& \mathbb{R}\\ \end{array} tq f(I) \subset I, la suite (U_n)définie par : 
    U_0 \in I \\  \forall n \; U_{n+1} = f(U_n) 
est appelé suite recurrente.


VI. Suites Complexes

Définition :

Une suite complexe est toute application d'une partie de \mathbb{N} à valeurs dans \mathbb{C} notée (Z_n) en général.
Remarques : 
1. Soit (Z_n) une suite complexe, alors il existe (x_n) et (y_n) \; \in \mathbb{R}^{\mathbb{N}} tq : \forall n \in \mathbb{N} \; : \; Z_n= x_n+i y_n et on note : \mathfrak{Re}(Z_n) = x_n et \mathfrak{Im}(Z_n)=y_n
2. Tous les résultats dans \mathbb{R} restent valables pour les suites complexes.
Proposition :

Soit (Z_n) une suite complexe, on a : (Z_n) est bornée \Leftrightarrow  (\mathfrak{Re}(Z_n) )_n et (\mathfrak{Im}(Z_n) )_n sont bornées.
Proposition :

Soit (Z_n) une suite complexe, et l = a+ib  \in \mathbb{C} avec a,b de \mathbb{R}
\displaystyle \lim \: Z_n = l \; \Leftrightarrow \; \displaystyle \lim\mathfrak{Re}(Z_n) = a et \displaystyle \lim\mathfrak{Im}(Z_n) = b .
Théorème de Bolzano - Weierstrass complexe :

De toute suite complexe bornée on peut extraire une suite convergente.